LSAT and Law School Admissions Forum

Get expert LSAT preparation and law school admissions advice from PowerScore Test Preparation.

 brittany1990
  • Posts: 21
  • Joined: Jul 11, 2012
|
#5913
Hi!

After reading the stimulus and question stem, I identified it as a weaken question. I wasn't sure exactly what to pre phrase, but I thought that I should look out for an answer choice that shows publishers now are not more interested in making money than in publishing books of intrinsic value.

I do not know why B is correct. I thought that this answer choice would STRENGTHEN the conclusion.

Thanks!
Brittany
 Steve Stein
PowerScore Staff
  • PowerScore Staff
  • Posts: 1153
  • Joined: Apr 11, 2011
|
#5915
Hey Brittany,

That one is a classic cause-effect stimulus. The author points out that traditionally, publishers have chosen to put out some books that had particular merit, in spite of the fact that those books were not likely to be profitable.

Fewer such meritorious books are published nowadays, however, and the author attributes this decrease to a greater interest on the part of publishers in making money.

Cause ..... ..... ..... :arrow: ..... ..... Effect
Publisher greed ..... :arrow: ..... Fewer good books

Since the stimulus is followed by a weaken question, the correct answer choice will weaken this causal hypothesis, in this case with an alternative cause:

If, as correct answer choice B provides, there has been a notable decline in high quality books, then that could be the cause of fewer good books being published. If this is the case, that would weaken the author's argument that the decrease is attributable to publisher greed.

I hope that's helpful! Let me know whether that clears this one up--thanks!

~Steve
 est15
  • Posts: 94
  • Joined: Aug 28, 2013
|
#15914
I thought the conclusion basically came down to publishers being more interested, now compared to before, in making money than in publishing books of intrinsic value. Thus, I thought (A) and (C) were valid choices because both suggest that the publishers were already interested in making money in the past, which would weaken the conclusion I described above. Since if there's proof that the publishers were already interested in making money in the past, it's harder to say that publishers now are more interested in making money than before. When I saw that (B) was the right answer, I felt like I was misunderstanding the conclusion and maybe even the argument. Can you explain what is going on in this passage and the answer choices? Thanks.
 Lucas Moreau
PowerScore Staff
  • PowerScore Staff
  • Posts: 216
  • Joined: Dec 13, 2012
|
#15923
Hello, est15,

I don't blame you for stumbling a little with this question - it's short and the answer choice doesn't go in the direction you'd expect, lol. :lol:

The conclusion is as you identify it, so good job there. Remember always that in Weaken questions, you're not just trying to weaken the conclusion itself, you're trying to weaken the link between premises and conclusion - you're trying to show that, even if all the premises are true, the conclusion does not logically follow from them.

So for answer choice A, that is more outright denying the conclusion than showing how the premise (less books with intrinsic merit are being published) does not lead to the conclusion (because publishers are now more interested in making money than they were before). They could very well have been always very interested, now they're just even more interested. :ras:

Answer choice C doesn't really apply, since it's talking about the past, and we're focused on the present. Also, the word "unexpectedly" is critical in showing how this is the wrong answer choice. It confirms that the motive of the publishers in publishing these books with merit was explicitly not money, since any money that was made from them was not expected. It also doesn't say this is no longer true in the present, which is what I believe it is trying to make you think it is saying. :hmm:

Answer choice B is the best, because it reveals an alternate cause for the decline in books with merit being published - less books with merit are being written. It has nothing to do with the motives of the publishers, they simply don't have as many high-quality books to publish as they did in years past. Therefore, it is showing that the premise does not necessarily lead to the conclusion.

Hope that helps,
Lucas Moreau

Get the most out of your LSAT Prep Plus subscription.

Analyze and track your performance with our Testing and Analytics Package.